Was passiert, wenn Sie versuchen, die Maxwell-Gleichungen auf dieses quantenmechanische System anzuwenden?

In einem anderen Beitrag haben wir die oszillierende Ladung in einem Wasserstoffatom diskutiert, und die Meinung schien zu sein, dass es tatsächlich eine oszillierende Ladung gibt, wenn man die Überlagerung der 1s- und 2p-Zustände betrachtet. Einer der Korrespondenten (freecharly) ging etwas weiter und sagte, dass Schrödinger diese oszillierende Ladung für die Strahlungsquelle hielt. Ich frage mich, ob die tatsächliche Berechnung dies bestätigt? Insbesondere im Fall des Wasserstoffatoms in dieser speziellen Überlagerung, erhalten Sie die korrekten Abklingzeiten für die Überlagerung von Zuständen, wenn Sie die Maxwell-Gleichungen auf die oszillierende Ladung anwenden und davon ausgehen, dass, wenn das System Energie durch Strahlung verliert, die "Wahrscheinlichkeit" fließt vom 2p- in den 1s-Zustand entsprechend der im System verbleibenden Energiemenge?

BEARBEITEN: Einige Leute lehnen die Grundvoraussetzung der Frage auf unterschiedliche Weise ab, also lassen Sie es mich etwas spezifischer machen: Ich frage nicht, ob Wasserstoffatome TATSÄCHLICH in einer bestimmten Überlagerung dieser Zustände EXISTIEREN. (Das kann ich in einer anderen Frage fragen.) Was ich hier frage, ist, WENN Sie (nur um genau zu sein) eine 50-50-Überlagerung der 1s- und 2p-Zustände nehmen und die Maxwell-Gleichungen auf die oszillierende Ladung anwenden UND das annehmen Wenn das Atom strahlt, sinkt die Wahrscheinlichkeit so vom angeregten Zustand in den Grundzustand, dass die Energieerhaltung erhalten bleibt ... Wenn Sie all diese Dinge tun, erhalten Sie ein Ergebnis, das mit der Standard-QM übereinstimmt?

Sie haben ein anderes Problem - wenn Sie nur auf dem initialisieren 2 P Zustand, der Ihnen die stärkste Emission geben sollte, gibt es keine Ladungsschwingung und daher keine Strahlung. (In ähnlicher Weise, wenn Sie beispielsweise initialisieren, 0,01 | 1 S + 0,99 | 2 P , die Oszillationsamplitude wird sehr klein sein und es wird lange dauern, bis sie auf Touren kommt.) Allerdings würde sich diese Frage ein wenig verbessern, wenn Sie den Mechanismus angeben, den Sie im Sinn haben, "wenn das System Energie verliert". Wahrscheinlichkeit fließt vom 2p- in den 2s-Zustand". Gehen Sie nur von einer Schrödinger-Dynamik aus? Oder etwas anderes?
Der strittige Punkt, Marty, ist, dass sich das Wasserstoffatom nicht wirklich in einer Überlagerung von 1s- und 2p-Zuständen befindet, es befindet sich in keinem der beiden Zustände. Es ist vergleichbar mit einem 1s-Zustand, der sich mit einer Energie von 10,2 EV hin und her bewegt, aber es ist eigentlich kein 1s-Zustand. In ähnlicher Weise ist es auch kein 2p-Zustand.
Sie stellen eine Frage ("Ich frage mich, ob ..."), ohne sich um eine Antwort bemühen zu müssen. Für jemanden mit so hohem Ruf ist das ein schlechtes Beispiel für Neuankömmlinge. Außerdem ist dies keine Diskussionsseite.
Ich mache mir keine Sorgen um das Beispiel, das ich Neuankömmlingen gebe.
In der zeitabhängigen Störungstheorie wird zur Beschreibung der Übergangswahrscheinlichkeit zwischen den Energieeigenzuständen die Zeitabhängigkeit des Quadrats der überlagerten Energieeigenfunktionen (entsprechend dem oszillierenden Dipolmoment) zur Berechnung der Übergangswahrscheinlichkeit verwendet. Siehe QM-Lehrbücher.
Gibt die halbklassische Berechnung also die gleiche Antwort oder nicht? Poste eine Antwort, wenn du kannst.
@Marty Green - Die halbklassische Berechnung von Übergangswahrscheinlichkeiten unter einer externen Störung verwendet nicht explizit den Energieverlust oder -gewinn durch Strahlung. Sie gibt die Übergangswahrscheinlichkeit als Funktion der Zeit an. Auch gibt es keine halbklassische Theorie für die spontane Emission, abgesehen von der aus den Einstein-Koeffizienten abgeleiteten Wahrscheinlichkeit.
Als ich „die halbklassische Rechnung“ sagte, meinte ich nicht die aus den Lehrbüchern. Ich meinte den, den ich in der Frage umrissen habe. Spontane Emission hat damit nichts zu tun, weil ich die jeweilige Überlagerung angegeben habe. Und die von mir skizzierte Berechnung forderte ausdrücklich die Verwendung des Strahlungsverlusts bei der Anpassung der s vs p-Koeffizienten im Verlauf des Übergangs.
@Marty Green - Ich finde Ihre Frage interessant, aber ich kenne keine "halbklassische" Berechnung der Abklingzeit in Bezug auf den Energieverlust aufgrund der Emission elektromagnetischer Wellen. Im üblichen quantenmechanischen Bild stellt sich auch das Problem, welche Energie einer beliebigen Überlagerung zweier Energieeigenzustände zuzuordnen ist. Die Überlagerung mit komplexen Koeffizienten C 1 Und C 2 liefert nur die Wahrscheinlichkeiten | C 1 | 2 Und | C 2 | 2 zum Messen der Energien E 1 Und E 2 in diesem Überlagerungszustand.
@freecharly Ich freue mich, dass Sie meine Frage interessant finden. Ich hatte gehofft, dass meine Absicht klar war, dass die Berechnung unter der Annahme durchgeführt werden sollte, dass die IST-Energie der Überlagerung als |c1|2*E1 + |c2|2 *E2 angenommen werden sollte, wobei c1 auf Kosten von c2 wächst als die Strahlung trägt Energie vom System weg. Ich denke, es ist eine ziemlich einfache Berechnung, und es ist seltsam für mich, dass Sie, vorausgesetzt, Sie kennen sich auf diesem Gebiet aus, nicht wissen, dass jemand eine Berechnung dieser Art veröffentlicht hat.
@Marty Green - Ich verstehe Ihre Annahme. Es steht wahrscheinlich im Widerspruch zum orthodoxen QM. Ich bin kein professioneller Experte, weder in der Geschichte noch in der konventionellen Quantentheorie oder ihren alternativen Interpretationen. Aber ich denke, dass es alternative Interpretationen zur sogenannten "Kopenhagener Interpretation" geben könnte, die der Realität näher kommen könnten. Also bleibe ich in dieser Angelegenheit aufgeschlossen.
@MartyGreen Warum findest du es seltsam, dass es noch nie zuvor gemacht wurde? Dies ist kein natürliches Modell (weil es keinen Zerfall für einen reinen 2s-Zustand vorhersagt und einen vollständig ad hoc-Rückwirkungsmechanismus postuliert). Das ist in der Tat eine interessante Frage, und es gibt Bereiche, die ähnliche Modelle verwenden (ohne die Ad-hoc-Back-Aktion), was unter einigen geeigneten Bedingungen gerechtfertigt werden kann. Wenn ich Zeit habe, schreibe ich es auf.
Die Überlagerung eines 2s- mit einem 1s-Zustand ist ein rotierender Magnet, wenn die beiden Zustände entgegengesetzte Spins haben. So strahlt es klassisch.

Antworten (2)

Ich vermute, dass die Ergebnisse (zumindest ungefähr) korrekt sind, da Barut seine „Selbstfeld-Elektrodynamik“ entwickelt hat (siehe zB http://phys.lsu.edu/~jdowling/publications/Barut89b.pdf ) und Ergebnisse behauptet sehr nah an denen von QED. In der Eigenfeldelektrodynamik wird Strahlung durch die Ladungsdichte in Bezug auf die Wellenfunktion auf übliche Weise (für das Dirac-Feld) erzeugt.

Ich bin enttäuscht, dass niemand in dieser Diskussionsgruppe eine endgültige Antwort darauf posten konnte, ob die halbklassische Berechnung, die Maxwell-Gleichungen auf die quantenmechanisch schwingende Ladung anwendet, das richtige Ergebnis für die Emission von Strahlung aus einer Anregung liefert Wasserstoffatom. Ich schätze Achmetelis Verweis auf eine verwandte Veröffentlichung, aber er geht nicht direkt auf diese Frage ein. Ich werde diese Frage also nach bestem Wissen und Gewissen selbst beantworten müssen, indem ich eine Berechnung vom Typ "Rückseite des Umschlags" demonstriere.

Ich sagte, ich wollte die 50-50-Überlagerung der 1s- und 2p-Zustände betrachten. Also müssen wir zuerst das maximale Dipolmoment der Überlagerung kennen. Ich habe das Ergebnis auf dieser Website der University of Texas von Prof. Richard Fitpatrick gefunden. Ich glaube, ich interpretiere es richtig, wenn ich sage, dass die maximale Ladungsverschiebung 0,4 Angström beträgt (etwa 75% des Standardradius des Grundzustands).

Dann brauchen wir die Frequenz der Schwingung. Dies ist natürlich die Differenzfrequenz, die der Energiedifferenz von 10,5 eV der Zustände entspricht, oder 1,6 x 10^16 rad/sec.

Jetzt können wir die Beschleunigung berechnen. Der einfachste Weg, dies zu tun, besteht darin, so zu tun, als ob es sich um eine gleichmäßige kreisförmige Bewegung handelt, und w^2*r zu verwenden. Ich bekomme eine Beschleunigung von 10^22/m-sec^2. (Da es sich tatsächlich um eine harmonische Bewegung und nicht um eine kreisförmige Bewegung handelt, erhalten wir im Endergebnis einen Fehlerfaktor von 2.)

Jetzt setze ich diese Beschleunigung einfach in die Larmour-Formel ein. Die Larmour-Formel findet man überall im Internet, aber ich habe einfach alle physikalischen Konstanten in Zahlenwerte umgerechnet, und es kommt auf

   **Radiated Power  =   6 x 10^-54 a^2**

Sie sehen, wenn ich meinen Beschleunigungswert in diese Formel einsetze, erhalte ich eine Gesamtstrahlleistung von 6 x 10^-10 Watt. Dies teilen wir durch 2, um die harmonische Bewegung gegenüber der Kreisbewegung zu berücksichtigen.

Ist das die richtige Leistung? Um das herauszufinden, müssen wir in die „Übergangszeit“ umrechnen. Die Gesamtenergie des angeregten Zustands beträgt 10,5 (nennen wir es 10) eV, was 1,6 x 10^-18 Joule entspricht. Wenn wir die Energie durch die Leistung dividieren, erhalten wir die Lebensdauer des angeregten Zustands von nur etwa 5 Nanosekunden. Oder vielleicht irre ich mich in Bezug auf die Energie und sollte sie als Hälfte nehmen (wegen der Überlagerung), was mir dann eine Lebensdauer von 2,5 Nanosekunden geben würde. So ähnlich.

Dies ist vielleicht nicht genau, aber ich denke, es ist ziemlich im Stadion.

Kaum zu glauben, wie schnell die Downvotes auf diese Antwort kommen. Stimmt etwas mit meiner Physik nicht?
Inzwischen habe ich im Lehrbuch von D. Griffith „Introduction to Quantum Mechanics“ im Kapitel 9.3.2 Lifetime of an Excited State gefunden, dass es für den Fall eines harmonischen Oszillators eine Berechnung der Leistung der emittierten Strahlung gibt nach QM und nach klassischer Elektrodynamik.
Man sollte nicht vergessen, dass die "Lebensdauer", die für diesen Übergang auch auf der verlinkten Webseite mit 1,6ns angegeben wird, eigentlich die Zeit bis zu einer großen Zahl ist N 0 der angeregten H-Atome zu zerfallen ist N 0 / e . Diese Lebensdauer, die einen plötzlichen Quantensprung für den Übergang annimmt, ist nicht dieselbe wie die, die Sie mit der Betrachtung der emittierten Energie berechnen.
Ob es genau dasselbe Leben ist oder nicht, es ist ziemlich nah dran. Und der "plötzliche Quantensprung" ist natürlich ebenso eine Annahme wie meine kontinuierliche Strahlung. Beide Annahmen sind Modelle, die uns bei der Durchführung der Berechnung leiten, und keine scheint gegenüber der anderen experimentell beweisbar zu sein.
Eigentlich sollte es möglich sein, zwischen diesen Modellen experimentell zu unterscheiden. Wenn das klassische Strahlungsmodell richtig wäre, müssten alle Atome mit der gleichen Zeit zerfallen. Quantenmechanisch zerfallen manche viel früher und manche viel später. Das sieht man auch beim radioaktiven Zerfall. Während die Halbwertszeit den Zerfall einer großen Anzahl von Atomen auf die Hälfte angibt, gibt es viele Atome, die erst nach einer um Größenordnungen längeren Zeit zerfallen.
Sie müssen wirklich sorgfältig darüber nachdenken, wie Sie ein Experiment aufbauen würden, um diesen vermeintlichen Unterschied zu demonstrieren. Ich glaube nicht, dass du das kannst.
Ich habe kurz gegoogelt und diesen Artikel über Lebensdauermessungen angeregter Zustände gefunden: slac.stanford.edu/cgi-wrap/getdoc/slac-pub-14754.pdf
Diese Antwort zeigt, dass Sie Antworten in der Größenordnung erhalten, aber das ist nicht so unerwartet, wie Sie denken - beide sind (relativ) einfache Theorien mit wenigen Dimensionskonstanten, die auf relativ ähnliche Weise verwendet werden, also gibt es das alles nicht viel Raum für sie, anderer Meinung zu sein. Auf jeden Fall übersehen Sie den wichtigen Teil: Damit das Modell überhaupt von Nutzen ist, muss es einen exponentiellen Abfall direkt vorhersagen, und es ist nicht klar, dass dies möglich ist (weil die Schwingungsamplitude des Dipols mit abnehmender Geschwindigkeit abnimmt). Überlagerungsänderungen).
Ich weiß auch nicht, woher deine Enttäuschung kommt. Die gestellte Frage ist interessant, erfordert jedoch eine ziemlich umfangreiche Berechnung, die in der Literatur (und aus guten Gründen) kaum untersucht wird, sodass die Beantwortung dieser Frage einige Zeit in Anspruch nimmt.